A 2. 5 × 103 kg elevator carries a maximum load of 709. 1 kg. A constant frictional force of 4. 0 x 103 N resists the elevator's motion upward. What minimum power must the motor deliver to lift the fully loaded elevator at a constant speed 2. 10 m/s? The acceleration due to gravity is 9. 81. Answer in units of kW

Answers

Answer 1

The minimum power must the motor deliver to lift the fully loaded elevator at a constant speed 2. 10 m/s is 19.1 kW.

What is speed?

Speed is the rate an object or person is moving in a given direction. It is measured as distance (meters, feet, miles, etc.) per unit of time (seconds, minutes, hours, etc.). It is an important and fundamental characteristic of matter, as it determines the kinetic energy of an object. Speed is also a vector quantity, as it describes both magnitude and direction. Speed has general and special relativity implications as well, as relative motion affects the propagation of light and space-time.

Step 1: Calculate the net force on the elevator:

Fnet = Ffr – mg

Fnet = 4.0 x 103 N – (2.5 x 103 kg)(9.81 m/s²)

Fnet = 9.10 x 103 N

Step 2: Calculate the power required to lift the elevator:

P = Fnet x v

P = (9.10 x 103 N) (2.10 m/s)

P = 19.1 kW

To learn more about speed

https://brainly.com/question/30249508

#SPJ4


Related Questions

design a circuit that can scale and shift the voltage from the range of -8 v ~0v to the range of 0 ~ 5v.

Answers

To scale and shift the voltage from the range of -8V to 0V to the range of 0V to 5V, you can use an inverting amplifier circuit with specific resistor values.

Design a circuit to scale and shift voltage from the range of -8V to 0V to the range of 0V to 5V.

To design a circuit that can scale and shift the voltage from the range of -8V to 0V to the range of 0V to 5V, you can use an operational amplifier (op-amp) circuit known as an inverting amplifier. Here's the circuit design:

1. Connect the inverting input (-) of the op-amp to the ground (0V reference).

2. Connect a resistor (R1) between the inverting input (-) and the output of the op-amp.

3. Connect a feedback resistor (R2) between the output of the op-amp and the inverting input (-).

4. Connect the input voltage source (Vin) between the inverting input (-) and the non-inverting input (+) of the op-amp.

5. Connect a voltage divider consisting of two resistors (R3 and R4) between the supply voltage (Vcc) and ground. Take the output voltage (Vout) from the junction between R3 and R4.

The resistor values can be calculated based on the desired scaling and shifting factors. In this case, we want to scale the voltage from -8V to 0V to the range of 0V to 5V.

Here's a set of example resistor values for scaling the voltage:

- R1 = 5kΩ

- R2 = 10kΩ

- R3 = 10kΩ

- R4 = 10kΩ

With these resistor values, the circuit will scale and shift the input voltage range as desired.

Learn more about amplifier circuit

brainly.com/question/29508163

#SPJ11

16. when water freezes in a closed jar both its volume and its pressure increase,eventually bursting the jar. does this violate the second-order conditionpv < 0? explain.

Answers

Water freezing in a closed jar and causing it to burst does not violate the second-order condition (pv < 0). This is because when water freezes, it undergoes a phase transition from liquid to solid, causing its volume to increase due to the formation of a crystalline structure.

However, this condition does not apply to water in a closed jar when it freezes and eventually bursts the jar. This is because water is not a gas and does not behave like a gas. When water freezes, it undergoes a phase change from a liquid to a solid, which results in a decrease in volume. However, this decrease in volume is accompanied by an increase in pressure because water expands when it freezes.


In summary, the second-order condition does not apply to water in a closed jar when it freezes and eventually bursts the jar because it is not a result of adiabatic expansion or compression of a gas. Instead, it is a phase change of a liquid, which results in an increase in pressure as the volume decreases.

To know more about crystalline visit:-

https://brainly.com/question/30545056

#SPJ11

Your camera'$ zoom lens has an adjustable focal length ranging from 55 t0 150 mm. Part (a) What is the maximum power of the lens_ Pmax' in diopters?

Answers

The maximum power of the lens is 18.18 diopters.

The maximum power of a zoom lens can be calculated using the formula Pmax = 1000/f, where f is the minimum focal length of the lens in millimeters. In this case, the minimum focal length of the zoom lens is 55 mm, so the maximum power can be calculated as Pmax = 1000/55 = 18.18 diopters.

A zoom lens is a type of lens that allows the user to adjust the focal length to capture images at different distances. The focal length of a lens is the distance between the lens and the point where the light rays converge to form a sharp image. The maximum power of a lens refers to the maximum magnification that can be achieved with that lens.

In the case of the given zoom lens with a focal length range of 55 to 150 mm, the maximum power can be calculated as 18.18 diopters. This means that the lens can magnify the subject up to 18 times its original size, which can be useful in situations where a closer view is required.

To know more about the zoom lens, click here;

https://brainly.com/question/26149171

#SPJ11

Determine the moment of inertia of the composite area about the x axis. Set a = 420 mm, b = 140 mm, h = 110 mm and r = 75 mm. Enter the number that corresponds to the units of mm^4 (millimeters to the fourth power). You may use scientific notation as follows: 2000 can be written as 2E3 or 20E2, 35000 can be written as 3.5E4, etc.

Answers

The moment of inertia of the composite shape about the x-axis is 2.702*10^8 mm^4.To determine the moment of inertia of the composite area about the x-axis, we need to break down the shape into simpler shapes and use the parallel axis theorem.

We have a rectangle with dimensions a x h and a semicircle with radius r. The moment of inertia of the rectangle about the x-axis is (1/12)*a*h^3, and the moment of inertia of the semicircle about its diameter (which is parallel to the x-axis) is (1/4)*pi*r^4.
Using the parallel axis theorem, we need to find the distance between the centroid of the composite shape and the x-axis. The centroid of the rectangle is at a/2 and h/2, and the centroid of the semicircle is at (4r)/(3*pi) from the diameter. The distance between the centroids and the x-axis is h/2 for the rectangle and (r + h) for the semicircle, so the distance between the centroid of the composite shape and the x-axis is (h/2)*((a^2+4r^2)/(a+2r)).
Now we can use the parallel axis theorem to find the moment of inertia of the composite shape about the x-axis:
I = (1/12)*a*h^3 + (1/4)*pi*r^4 + (h/2)*((a^2+4r^2)/(a+2r))*(h/2)^2 + (1/2)*pi*r^2*(r + h - (4r)/(3*pi))^2
Plugging in the given values of a, b, h, and r, we get:
I = 2.702*10^8 mm^4
Therefore, the moment of inertia of the composite shape about the x-axis is 2.702*10^8 mm^4.
To know more about parallel axis theorem visit:

https://brainly.com/question/30460015

#SPJ11

how many times more massive is the sun than an asteroid which is y times more massive than the martian moon phobos?

Answers

The mass of the Sun is approximately 1.989 x 10^30 kg, while the mass of Phobos, the Martian moon, is about 1.0659 x 10^16 kg. If an asteroid is "y" times more massive than Phobos, its mass would be y * 1.0659 x 10^16 kg.

To provide a thorough answer, I would need a bit more information such as the exact value of y and the mass of Phobos. However, I can give a general formula that can be used to find the answer. Let's say the mass of Phobos is m and the asteroid is y times more massive than Phobos, then the mass of the asteroid would be ym.

Now, to find how many times more massive the sun is compared to the asteroid, we can divide the mass of the sun (which is approximately 1.989 x 10^30 kg) by the mass of the asteroid (which is ym). So the formula would be:
mass of sun / mass of asteroid = 1.989 x 10^30 kg / ym
Simplifying this equation, we get:
mass of sun / mass of asteroid = (1.989 x 10^30) / (y x m)
The mass of the Sun is approximately 1.989 x 10^30 kg, while the mass of Phobos, the Martian moon, is about 1.0659 x 10^16 kg. If an asteroid is "y" times more massive than Phobos, its mass would be y * 1.0659 x 10^16 kg.
To find out how many times more massive the Sun is than the asteroid, divide the mass of the Sun by the mass of the asteroid:
Sun's mass / Asteroid's mass = (1.989 x 10^30 kg) / (y * 1.0659 x 10^16 kg)
Simplify the expression:
(1.989 x 10^30) / (y * 1.0659 x 10^16)
Thus, the Sun is (1.989 x 10^30) / (y * 1.0659 x 10^16) times more massive than the asteroid.

To know more about mass visit:-

https://brainly.com/question/18064917

#SPJ11

A bottle contains a gas with atoms whose lowest four energy levels are -12 eV, -9 eV, -8 eV, and -2 eV. Electrons run through the bottle and excite the atoms so that at all times there are large numbers of atoms in each of these four energy levels, but there are no atoms in higher energy levels. List the energies of the photons that will be emitted by the gas. Give the lowest photon energy first and the highest photon energy last:

Answers

The correct order will be 1 eV, 3 eV, 4 eV, 6 eV, 7 eV, and 10 eV.

The photons emitted will be due to the atoms transitioning from higher energy levels to lower energy levels.

The energy of a photon is given by E = hf, where h is Planck's constant and f is the frequency of the photon.

The frequency can be calculated as the energy difference between the two levels divided by Planck's constant.

Using this formula, the energies and corresponding frequencies of the photons emitted by the gas are:

- From -2 eV to -8 eV: E = |-2 eV - (-8 eV)| = 6 eV, f = 6 eV / h

- From -2 eV to -9 eV: E = |-2 eV - (-9 eV)| = 7 eV, f = 7 eV / h

- From -2 eV to -12 eV: E = |-2 eV - (-12 eV)| = 10 eV, f = 10 eV / h

- From -8 eV to -12 eV: E = |-8 eV - (-12 eV)| = 4 eV, f = 4 eV / h

- From -9 eV to -12 eV: E = |-9 eV - (-12 eV)| = 3 eV, f = 3 eV / h

- From -8 eV to -9 eV: E = |-8 eV - (-9 eV)| = 1 eV, f = 1 eV / h

Arranging the energies in increasing order, the photons emitted will have energies of 1 eV, 3 eV, 4 eV, 6 eV, 7 eV, and 10 eV.

To know more about photons, refer here

https://brainly.com/question/32069452#

#SPJ11

a step-up transformer is designed to produce 1840 v from a 115-v ac source. if there are 384 turns on the secondary coil, how many turns should be wound on the primary coil?

Answers

A step-up transformer is designed to produce 1840 v from a 115-v ac source. if there are 384 turns on the secondary coil, the number of turns required on the primary coil of the step-up transformer is 24.

To determine the number of turns required on the primary coil of a step-up transformer, we can use the turns ratio equation:

Turns ratio = (Number of turns on secondary coil) / (Number of turns on primary coil)

Given:

Voltage on the secondary coil ([tex]V_secondary[/tex]) = 1840 V

Voltage on the primary coil ([tex]V_primary[/tex]) = 115 V

Number of turns on the secondary coil ([tex]N_secondary[/tex]) = 384

We need to solve for the number of turns on the primary coil ([tex]N_primary[/tex]).

Using the turns ratio equation:

Turns ratio = [tex]V_secondary[/tex] / [tex]V_primary[/tex] = [tex]N_secondary[/tex] / [tex]N_primary[/tex]

Plugging in the given values:

1840 V / 115 V = 384 / [tex]N_primary[/tex]

Simplifying the equation:

16 = 384 / [tex]N_primary[/tex]

To solve for [tex]N_primary[/tex], we can rearrange the equation:

[tex]N_primary[/tex] = 384 / 16

[tex]N_primary[/tex] = 24

For more such information on: coil

https://brainly.com/question/23935748

#SPJ11

find an equation of the line that satisfies the given conditions

Answers

The equation of a line is commonly represented as y = mx + b, where y is the dependent variable (usually representing the vertical axis), x is the independent variable (usually representing the horizontal axis), m is the slope of the line, and b is the y-intercept.

The slope (m) of a line determines its steepness or inclination. It represents the rate of change of the dependent variable (y) with respect to the independent variable (x). A positive slope indicates an upward trend, while a negative slope indicates a downward trend.

The y-intercept (b) is the point where the line intersects the y-axis. It represents the value of y when x is equal to zero. It gives us a starting point on the y-axis for the line.

By knowing the slope and y-intercept, we can substitute their values into the equation y = mx + b to obtain the specific equation of the line that satisfies the given conditions. This equation allows us to determine the value of y for any given value of x along the line.

Learn more about vertical axis here:

https://brainly.com/question/31086219

#SPJ11

A 0.500 kg toy car moves in a circular path of radius 1.50 m at 1.2 m/s. 6a. What are the period and frequency of the circular motion? 27 2 Frequency 5b. What are the centripetal acceleration and centripetal force Centripetal acceleration a my Centripetal force 5c. What would the velocity have to be in order to require twice the centripetal force? velocity V m 5d. If the velocity in part a is doubled, how much centripetal force is required Centripetal force to keep the car in circular motion?

Answers

The period and the frequency of the circular  motion is 7.85 sec and 0.127Hz respectively. The centripetal acceleration is  [tex]0.96 m/s^2[/tex] and centripetal force is 0.48 N.

a) The period of the circular motion can be calculated using the formula:

[tex]T =\frac{2\pi r}{v}[/tex]

where r is the radius of the circular path and v is the speed of the toy car. Substituting the given values, we get:

[tex]T = \frac{2\pi (1.50 m)}{1.2}[/tex]  = 7.85 s

Therefore, the period of the circular motion is approximately 7.85 seconds.

The frequency of the circular motion is the reciprocal of the period:

f = [tex]\frac{1}{T}[/tex] = 0.127 Hz

Therefore, the frequency of the circular motion is approximately 0.127 hertz.

b) The centripetal acceleration of the toy car can be calculated using the formula:

a =[tex]\frac{v^2}{r}[/tex]

where v is the speed of the toy car and r is the radius of the circular path. Substituting the given values, we get:

a = [tex](1.2 m/s)^2/(1.50 m)[/tex] = [tex]0.96 m/s^2[/tex]

Therefore, the centripetal acceleration of the toy car is approximately  [tex]0.96 m/s^2[/tex]

The centripetal force required to keep the toy car in circular motion can be calculated using the formula:

F = ma

where m is the mass of the toy car and a is the centripetal acceleration. Substituting the given values, we get:

F = (0.500 kg) × (0.96 [tex]m/s^2[/tex]) = 0.48 N

Therefore, the centripetal force required to keep the toy car in circular motion is approximately 0.48 newtons.

c) If the centripetal force required to keep the toy car in circular motion is doubled, the velocity of the toy car must be increased. We can use the centripetal force formula to solve for the required velocity:

F = ma = [tex]mv^2/r[/tex]

If we double the centripetal force, we get:

2F = [tex]mv^2/r[/tex]

Solving for v, we get:

v = [tex]\sqrt[]{(2Fr/m)}[/tex]

Substituting the given values, we get:

v = [tex]\sqrt[]{(2)(0.48 N)(1.50 m)/(0.500 kg))}[/tex] =  1.72 m/s

Therefore, the velocity of the toy car would need to be approximately 1.72 meters per second to require twice the centripetal force.

d) If the velocity of the toy car is doubled, the centripetal force required to keep the car in circular motion will increase four times. We can use the centripetal force formula to calculate the new force:

F' = [tex]mv'^2/r[/tex]= [tex]m(2v)^2/r[/tex]= [tex]4mv^2/r[/tex]

Substituting the given values, we get:

F' = (0.500 kg)×(4)×(1.2 [tex]m/s)^2[/tex]/(1.50 m) = 1.92 N

Therefore, the centripetal force required to keep the toy car in circular motion when the velocity is doubled is approximately 1.92 newtons.

To know more about circular Motion

https://brainly.com/question/14625932

#SPJ4

what is the magnitude of the net force

Answers

Answer:

←13

Explanation:

You see the two men pushing it the opposite direction from each other. When you see different net forces you subtract the numbers from each other. So 53- 40=13. You see which value holds the greatest amount and say that the answer you got is supposed to be that side of the net force. The answer is "13 net force to the left, OR ←13"

an ideal spring has a spring constant (force constant) of 2500 n/m. how much work is required to stretch the spring by 2.0 cm?

Answers

The work required to stretch the spring having a spring constant (force constant) of 2500 n/m by 2.0 cm is 0.2 Joules.

The work required to stretch an ideal spring can be calculated using the formula:

Work = [tex](1/2) * k * x^2[/tex]

Where k is the spring constant and x is the displacement from the equilibrium position.

Given that the spring constant is 2500 N/m and the displacement is 2.0 cm (or 0.02 m), we can substitute these values into the formula:

Work =[tex](1/2) * 2500 N/m * (0.02 m)^2[/tex]

Calculating this expression, we get:

[tex]Work = (1/2) * 2500 N/m * 0.0004 m^2 \\Work = 0.5 N * 0.0004 m^2[/tex]

Work = 0.0002 Nm = 0.2 J

Therefore, the work required to stretch the spring by 2.0 cm is 0.2 Joules.

To know more about equilibrium position, here

brainly.com/question/31576470

#SPJ4

How would you show that RC has units of seconds if R is measured in ohms and C is measured in farads

Answers

RC has the units of seconds (s).

To show that RC has units of seconds when R is measured in ohms and C is measured in farads, you simply multiply the units of R and C together:

R (resistance) is measured in ohms (Ω)
C (capacitance) is measured in farads (F)

Now multiply these units:

RC = Ω × F

Since 1 Ω = 1 kg·m²·s⁻³·A⁻² and 1 F = 1 s⁴·A²·m⁻²·kg⁻¹, we can replace the units:

RC = (kg·m²·s⁻³·A⁻²) × (s⁴·A²·m⁻²·kg⁻¹)

Now, cancel out the common terms (kg, A, and m):

RC = s

As a result, RC has the units of seconds (s).

To learn more about resistance, refer below:

https://brainly.com/question/30799966

#SPJ11

Explain what it means for the radial velocity signature of an exoplanet to be periodic. Why is the signature periodic?

Answers

The periodicity of the radial velocity signal offers useful information on the orbit, mass, and other features of the exoplanet and is an important technique for discovering and characterising exoplanets.

The radial velocity signature of an exoplanet refers to the periodic changes in the velocity of its host star, caused by the gravitational tug of the planet as it orbits around the star. Specifically, the radial velocity signature is the variation in the star's velocity along the line of sight of an observer on Earth, as measured by the Doppler effect.

When a planet orbits a star, both the star and the planet orbit around their common center of mass. The gravitational pull of the planet causes the star to move in a small circular or elliptical orbit, with the star's velocity changing as it moves towards or away from the observer on Earth.

The velocity change of the star can be detected using the Doppler effect, which causes the star's spectral lines to shift towards the blue or red end of the spectrum, depending on whether the star is moving towards or away from the observer. By measuring these velocity shifts over time, astronomers can determine the period, amplitude, and other properties of the exoplanet's orbit.

If the radial velocity signature of an exoplanet is periodic, it means that the changes in the star's velocity occur at regular intervals, corresponding to the planet's orbital period. This periodicity arises from the fact that the planet orbits the star in a regular, predictable way, and exerts a gravitational pull on the star that varies in strength over time as the planet moves closer or further away.

Overall, the periodicity of the radial velocity signature provides valuable information about the exoplanet's orbit, mass, and other properties, and is an important tool for detecting and characterizing exoplanets.

Learn more about radial velocity on:

https://brainly.com/question/28044471

#SPJ11

The following data of position x and time t are collected for an object that starts at rest and moves with constant acceleration. t(s) x(m) 0 2 1 5 2 14 3 29 The position of the object at t = 5s is most nearly A. 30m B. 45m C. 75m D. 77m E. 110m

Answers

The position of the object at t = 5s is most nearly is 75m. The correct option is C.

In this question, we are given the position x and time t data for an object that starts at rest and moves with constant acceleration. The acceleration is constant because the change in position is proportional to the square of time. We can use the formula x = ut + 1/2at², where u is the initial velocity (which is zero in this case), a is the acceleration, and t is the time, to find the acceleration of the object.

Using the given data, we can calculate the acceleration as follows:

x = ut + 1/2at²

When t = 1s, x = 2m
When t = 2s, x = 5m
When t = 3s, x = 14m

Substituting these values in the formula, we get:

2 = 0 + 1/2a(1)²
5 = 0 + 1/2a(2)²
14 = 0 + 1/2a(3)²

Solving for a, we get a = 6m/s².

Now, we can use the formula x = ut + 1/2at² to find the position of the object at t = 5s.

x = 0 + 1/2(6)(5)²
x = 75m

Therefore, the position of the object at t = 5s is most nearly C. 75m.

To know more about acceleration, refer to the link below:

https://brainly.com/question/13105620#

#SPJ11

Laser Cooling Lasers can cool a group of atoms by slowing them down, because the slower the atoms, the lower their temperature. A rubidium atom of mass 1.42×10−25kg and speed 229 m/s undergoes laser cooling when it absorbs a photon of wavelength 781 nm that is traveling in a direction opposite to the motion of the atom. This occurs a total of 7700 times in rapid succession. Part A What is the atom’s new speed after the 7700 absorption events? WRITE CORRECT UNITS Part B How many such absorption events are required to bring the rubidium atom to rest from its initial speed of 229 m/s? Express your answer to three significant figures.

Answers

atom’s new speed after the 7700 absorption events is 214.3 m/s and and number of such events required to bring rubidium atom to rest from its initial speed of 229 m/s are 111.7


Part A:
To find the new speed of the atom after 7700 absorption events, we need to use the formula:

Δv = (h/λ) * (Γ/2) * (S / (1 + S + 4Δ²/Γ²))

Where:
h = Planck's constant = 6.626 x 10^-34 J*s
λ = wavelength of the photon = 781 nm = 7.81 x 10^-7 m
Γ = natural linewidth of the rubidium atom = 6.07 x 10^6 s^-1
S = saturation parameter = 2.64 x 10^7
Δ = detuning parameter = -1.5 x 10^9 Hz

Plugging in these values, we get:

Δv = (6.626 x 10^-34 J*s / 7.81 x 10^-7 m) * (6.07 x 10^6 s^-1 / 2) * (2.64 x 10^7 / (1 + 2.64 x 10^7 + 4(-1.5 x 10^9)^2/(6.07 x 10^6)^2))

Δv = -2.05 m/s (rounded to two significant figures)

Therefore, the atom's new speed after 7700 absorption events is:

229 m/s - 7700 * 2.05 m/s = 214.3 m/s

Answer: 214.3 m/s

Part B:
To find the number of absorption events required to bring the rubidium atom to rest, we need to find the total change in velocity that is needed. Since the final velocity is zero, the total change in velocity is equal to the initial velocity. Therefore:

Total change in velocity = 229 m/s

Using the same formula as in Part A, we can find the change in velocity per absorption event:

Δv = (h/λ) * (Γ/2) * (S / (1 + S + 4Δ²/Γ²))

Plugging in the same values as in Part A, we get:

Δv = -2.05 m/s

To find the number of absorption events required, we can divide the total change in velocity by the change in velocity per event:

Number of absorption events = Total change in velocity / Δv

Number of absorption events = 229 m/s / 2.05 m/s

Number of absorption events = 111.7

Rounding to three significant figures, we get:

Answer: more than 100 (111.7 rounded)


to know more about cooling laser

brainly.com/question/11742815

#SPJ11

A proton of energy 900GeV collides with a stationary proton. Find the available energy Ea. The rest energy of the proton is 938MeV. Express your answer in billions of electron volts to two significant figures.
A proton and an antiproton have equal energies of 450GeV. The particles collide head-on. Find the available energy Ea. The rest energy of the proton is 938MeV. Express your answer in billions of electron volts to two significant figures.

Answers

The rest energy of the proton is 938MeV is Ea = E - 2E0 = 1.797 x 10^11 eV and The total available energy is Ea = E - 2E0 = 8.998 x 10^10 eV.

For the first question, we can use the conservation of energy and momentum to find the available energy Ea. Since one proton is stationary, its momentum is zero. The momentum of the other proton can be found using the equation p = mv, where p is the momentum, m is the mass, and v is the velocity. The velocity of the proton can be found using the equation E = mc^2, where E is the energy, m is the mass, and c is the speed of light. Therefore, the velocity of the proton is v = c * sqrt(1 - (m*c^2/E)^2), where m is the rest energy of the proton and E is the energy of the proton. Plugging in the given values, we get v = 0.9999999968c. The momentum of the proton is then p = mv = 8.99111 x 10^-19 kg m/s. The total energy of the system is E = 2E0 + Ea, where E0 is the rest energy of the proton. Therefore, Ea = E - 2E0 = 1.797 x 10^11 eV. Rounded to two significant figures, the answer is 180 billion electron volts.


For the second question, we can again use the conservation of energy and momentum. Since the particles have equal energies, they have equal momenta. The total energy of the system is E = 2E0 + Ea, where E0 is the rest energy of the proton and Ea is the available energy. Using the same equation as before, we can find that the velocity of the particles is v = c * sqrt(1 - (m*c^2/E)^2), where m is the rest energy of the proton and E is the energy of the particles. Plugging in the given values, we get v = 0.9999999783c. The momentum of each particle is then p = mv = 4.5007 x 10^-19 kg m/s. The total available energy is Ea = E - 2E0 = 8.998 x 10^10 eV. Rounded to two significant figures, the answer is 90 billion electron volts.

To know more about Proton visit:

https://brainly.com/question/1252435

#SPJ11

An LRC series circuit has R = 15.0 ?, L = 25.0 mH, and C = 30.0 ?F. The circuit is connected to a
120-V (rms) ac source with frequency 200 Hz.
(a) What is the impedance of the circuit?
(b) What is the rms current in the circuit?
(c) What is the rms voltage across the resistor?
(d) What is the rms voltage across the inductor?
(e) What is the rms voltage across the capacitor?

Answers

The impedance of the circuit is 14.8 ohms. The current amplitude in the circuit is 8.11 A, and the phase angle between the current and voltage in the circuit is 0.542 radians.

The impedance of an LRC series circuit is given by Z = R + j(XL - XC), where R is the resistance, XL is the inductive reactance, and XC is the capacitive reactance. The inductive and capacitive reactances are given by XL = ωL and XC = 1/(ωC), respectively. The impedance of the circuit is calculated to be 14.8 ohms. The current amplitude in the circuit is calculated using Ohm's law as I = V/Z, where V is the voltage amplitude of the source. The current amplitude is found to be 8.11 A. The phase angle between the current and voltage in the circuit is calculated using the arctan function of the ratio of the imaginary part of the impedance to the real part of the impedance. The phase angle is found to be 0.542 radians, which indicates that the current is leading the voltage in the circuit by this amount.

Learn more about impedance:

https://brainly.com/question/2263607

#SPJ11

if star a is closer to us than star b, then star a's parallax angle is _________. larger than that of star b smaller than that of star b fewer parsecs than that of star b hotter than that of star b

Answers

The correct answer is Smaller than that of star B.

The parallax angle of a star is inversely proportional to its distance from Earth. Therefore, if star A is closer to us than star B, star A's parallax angle will be smaller than that of star B. The parallax angle is a measure of the apparent shift of a star's position when viewed from different vantage points on Earth's orbit.

Parallax is used to determine the distance to nearby stars. By measuring the parallax angle of a star, astronomers can calculate its distance using trigonometric principles. The smaller the parallax angle, the greater the distance to the star.

In the context of the question, since star A is closer to us than star B, it means that star A is at a shorter distance from Earth. Consequently, its parallax angle will be smaller compared to the parallax angle of star B, which indicates that star A is farther away from us than star B.

Learn more about Parallax angle here: https://brainly.com/question/20296211

#SPJ11

A current of 4. 75 A is going through a 5. 5 mH inductor is switched off. It takes 8. 47 ms for the current to stop flowing.

> What is the magnitude of the average induced emf, in volts, opposing the decrease of the current?

Answers

A current of 4.75 A is going through a 5.5 mH inductor is switched off. It takes 8.47 ms for the current to stop flowing. The average induced emf opposing the decrease of the current is approximately 26.125 volts.

To calculate the magnitude of the average induced electromotive force (emf) opposing the decrease of the current, we can use Faraday's law of electromagnetic induction.

Faraday's law states that the induced emf in an inductor is equal to the rate of change of magnetic flux through the inductor. Mathematically, it can be expressed as:

emf = -L * (di/dt)

Where:

emf is the induced emf (in volts)

L is the inductance of the inductor (in henries)

di/dt is the rate of change of current (in amperes per second)

Given:

Current (I) = 4.75 A

Inductance (L) = 5.5 mH = 5.5 x [tex]10^{-3}[/tex] H

Time (t) = 8.47 ms = 8.47 x [tex]10^{-3}[/tex]) s

To find di/dt, we need to calculate the change in current over time. Since the current is decreasing to zero, di will be the initial current minus the final current, and dt will be the time taken for the current to decrease.

Initial current (Iinitial) = 4.75 A

Final current (Ifinal) = 0 A

di = Iinitial - Ifinal = 4.75 A - 0 A = 4.75 A

dt = 8.47 x [tex]10^{-3}[/tex] s

Now we can calculate the magnitude of the average induced emf:

emf = -L * (di/dt)

= - (5.5 x [tex]10^{-3}[/tex] H) * (4.75 A / 8.47 x [tex]10^{-3}[/tex] s)

Calculating the value:

emf = - (5.5 x [tex]10^{-3}[/tex] H) * (4.75 A / 8.47 x [tex]10^{-3}[/tex] s)

= - (5.5 x [tex]10^{-3}[/tex]) H) * (4.75 A / 8.47 x [tex]10^{-3}[/tex] s)

= - (5.5 x 4.75) V

= - 26.125 V

Taking the magnitude, the average induced emf opposing the decrease of the current is approximately 26.125 volts.

To know more about average induced emf here

https://brainly.com/question/31979754

#SPJ4

the alpha particles emitted by radon–222 have an energy of 8.8 × 10–13 j. if a 200. g pb brick absorbs 1.0 × 1010 alpha particles from radon decay, what dose in rads will the brick absorb?

Answers

The brick will absorb 0.044 rads of radiation dose.

Radon decay alpha particles absorbed, dose?

To calculate the dose in rads absorbed by the brick, we can use the following formula:

dose (in rads) = energy absorbed (in joules) / mass of absorbing material (in kg)

First, we need to calculate the energy absorbed by the brick. The energy of one alpha particle is given as 8.8 × [tex]10^-^1^3[/tex]J. Therefore, the total energy absorbed by 1.0 × 1010 alpha particles is:

energy absorbed = (8.8 × [tex]10^-^1^3[/tex]J/alpha particle) x (1.0 × [tex]10^1^0[/tex] alpha particles) = 8.8 × [tex]10^-^3[/tex] J

Now, we can calculate the dose in rads absorbed by the brick:

dose = (8.8 × [tex]10^-^3[/tex] J) / (0.200 kg) = 0.044 rads

Learn more about radiation

brainly.com/question/1497235

#SPJ11

true/false. the angular momentum about the center of the planet and the total mechanical energy will be conserved regardless of whether the object moves from small rrr to large rrr

Answers

Answer:

Total Energy = Kinetic Energy + Potential Energy

The total energy will be a constant since no external forces are present

KE and PE can each shift from one form to the other with the total energy remaining constant

a random sample of 15 college soccer players were selected to investigate the relationship between heart rate and maximal oxygen uptake. the heart rate and maximal oxygen uptake were recorded for each player during a training session. a regression analysis of the data was conducted, where heart rate is the explanatory variable and maximal oxygen uptake is the response variable.

Answers

A regression analysis was conducted on heart rate and maximal oxygen uptake data for 15 college soccer players to investigate their relationship during a training session.

In the study, a random sample of 15 college soccer players were selected to investigate the relationship between heart rate and maximal oxygen uptake. Heart rate and maximal oxygen uptake were recorded for each player during a training session. A regression analysis was conducted to model the relationship between heart rate (independent variable) and maximal oxygen uptake (dependent variable). The regression equation can be used to predict maximal oxygen uptake for a given heart rate. The analysis also provides information about the strength and direction of the relationship between the two variables. This study can provide valuable insights into the relationship between heart rate and maximal oxygen uptake in college soccer players and may have implications for training and performance strategies.

Learn more about regression analysis here:

https://brainly.com/question/30011167

#SPJ11

. a near-sighted person can only see objects clearly up to a maximum distance dmax. design a lens to correct the vision of a person for whom dmax = 37 cm.

Answers

We would need to find a concave lens with a power of -0.37 diopters and place it in front of the person's eye. This lens would diverge the incoming light rays and reduce the refractive power of the eye, allowing the light to focus correctly on the retina and correcting the person's near-sightedness.

To correct the vision of a near-sighted person with a maximum clear distance of 37 cm, we need to design a concave lens that will diverge the light rays before they enter the eye, so that they will focus correctly on the retina.

The strength of the lens required to correct the vision depends on the refractive power of the eye, which is measured in diopters. A near-sighted person has too much refractive power, which causes the light rays to focus in front of the retina, resulting in a blurry image.

To correct this, we need to add a negative lens (concave lens) in front of the eye that will reduce the total refractive power. The strength of the lens needed can be calculated using the formula:

Lens power (in diopters) = 1 / focal length (in meters)

Since the person can only see clearly up to a distance of 37 cm, the focal length of the lens needed is:

focal length = 1 / (dmax / 100) = 1 / 0.37 = 2.70 meters

Therefore, the lens power required to correct the near-sightedness is:

Lens power = 1 / focal length = 1 / 2.70 = 0.37 diopters

For more such questions on refractive power

https://brainly.com/question/25164545

#SPJ11

To correct the vision of a near-sighted person who can only see objects clearly up to a maximum distance of d max = 37 cm, a concave lens would be required.

This type of lens diverges light rays and causes them to spread out, which corrects the near-sightedness. The strength of the lens would need to be calculated based on the distance of the object that the person wants to see clearly. For example, if the person wants to see an object at a distance of 50 cm, a lens with a strength of -2.5 diopters would be needed. It is important to note that the lens can only correct vision up to a certain point, and the person may still need to wear corrective lenses for distant vision beyond their dmax.
To design a lens to correct the vision of a near-sighted person with a maximum clear distance (dmax) of 37 cm, follow these steps:
1. Identify the person's maximum clear distance: In this case, dmax = 37 cm.
2. Determine the focal length (f) needed to correct their vision: Use the formula 1/f = 1/dmax. In this case, 1/f = 1/37 cm.
3. Calculate the focal length (f): Solve the equation from step 2 to find f. In this case, f = 37 cm.
4. Choose a lens with a negative focal length: Since the person is near-sighted, you'll need a diverging lens with a negative focal length. In this case, choose a lens with a focal length of -37 cm.
To summarize, to correct the vision of a person with a dmax of 37 cm, you would need a diverging lens with a focal length of -37 cm. This lens will help the person see objects clearly at a greater distance.

Visit here to learn more about near-sighted person:

brainly.com/question/31606655

#SPJ11

that factors other than the relative motion between the source and the observer can influence the perceived frequency change

Answers

The factors in the Doppler effect on which the change in frequency depends includes: Medium, source characteristics, Observer motion, and Reflecting surfaces.

How do we explain?

The Doppler effect describes the result of waves coming from a moving source. There appears to be an upward shift in frequency for observers facing the source, whereas there appears to be a downward shift for observers facing away from the source.

The Doppler effect causes a source's received frequency—how it is perceived when it arrives at its destination—to differ from the broadcast frequency when there is motion that increases or decreases the distance between the source and the receiver.

Learn more about Doppler effect at:

https://brainly.com/question/28106478

#SPJ1

#complete question:

Name the factors in the Doppler effect on which the change in frequency depends.

a pulse of radiation propagates with velocity vector v = < 0, 0, −c >. the electric field in the pulse is vector e = < 7.7 ✕ 106, 0, 0 > n/c. what is the magnetic field in the pulse?

Answers

A pulse of radiation propagates with velocity vector v = < 0, 0, −c >. The electric field in the pulse is vector e = < 7.7 ✕ 106, 0, 0 > n/c. The magnetic field in the pulse is B = < 7.7 ✕ 106t, 0, 0 > n/c

To find the magnetic field in the pulse, we can use the Maxwell's equations:

curl(E) = -dB/dt

where E is the electric field and B is the magnetic field.

Since the electric field is given as e = < 7.7 ✕ 106, 0, 0 > n/c and the velocity vector is v = < 0, 0, −c >, we can assume that the pulse is propagating in the negative z-direction.

Therefore, we can write the electric field as:

e = < 0, 0, 7.7 ✕ 106 > n/c

Now, we can use the Maxwell's equation to find the magnetic field:

curl(E) = -dB/dt

Taking the curl of the electric field, we get:

curl(E) = < 0, -7.7 ✕ 106, 0 > n/c

Since the pulse is propagating in the negative z-direction, we can assume that the magnetic field is only in the x-direction. Therefore, we can write the magnetic field as:

B = < Bx, 0, 0 >

Now, substituting the values of curl(E) and B in Maxwell's equation, we get:

< 0, -7.7 ✕ 106, 0 > = -dBx/dt

Integrating both sides with respect to time, we get:

Bx = 7.7 ✕ 106t + C

where C is a constant of integration.

Since the magnetic field is zero at t = 0, we can assume that C = 0. Therefore, the magnetic field in the pulse is:

B = < 7.7 ✕ 106t, 0, 0 > n/c

To know more about magnetic field refer here:

https://brainly.com/question/12970526#

#SPJ11

Which of the following statements correctly describes the change which occurs when a liquid vaporizes at its boiling point at a given external pressure?
a) The entropy decreases.
b) The temperature increases.
c) The kinetic energy increases.
d) The potential energy increases.

Answers

When a liquid vaporizes at its boiling point at a given external pressure, the correct statement that describes the change is that the kinetic energy increases. Option c.

This is because as the liquid is heated to its boiling point, the temperature remains constant until all of the liquid has vaporized. During this phase change, the energy supplied to the liquid is used to break the intermolecular forces between the liquid particles, increasing their kinetic energy and causing them to escape into the gas phase. The entropy of the system also increases, as the liquid molecules are now more disordered in the gas phase than they were in the liquid phase.

The potential energy of the system remains constant during this process, as there is no change in the distance between the particles. Therefore, the correct statement is that the kinetic energy increases when a liquid vaporizes at its boiling point at a given external pressure. Answer option c.

More on pressure: https://brainly.com/question/21039613

#SPJ11

Determine the maximum flow rate (kg/s) and corresponding pressure gradient (Pa/m) for which laminar flow would occur for water, SAE 10W oil, and glycerin. The fluids are at 20 deg. C. Draw some conclusion from your analysis.

Answers

Pressure gradient is directly proportional to the flow rate for each fluid. Fluids with higher viscosities require larger pressure gradients to achieve the same flow rates, indicating a higher resistance to flow.

To determine the maximum flow rate and corresponding pressure gradient for laminar flow, we can use the Hagen-Poiseuille equation, which relates the flow rate to the pressure gradient for viscous flow in a cylindrical pipe:

Q = (π * ΔP * [tex]r^{4}[/tex]) / (8 * μ * L),

where Q is the flow rate, ΔP is the pressure gradient, r is the radius of the pipe, μ is the dynamic viscosity of the fluid, and L is the length of the pipe.

We can rearrange this equation to solve for the pressure gradient:

ΔP = (8 * μ * Q) / (π  * [tex]r^{4}[/tex] * L).

Given that the fluids are water, SAE 10W oil, and glycerin at 20°C, we can look up their respective dynamic viscosities at this temperature:

Water: μwater = 0.001 kg/(m·s)

SAE 10W oil: μoil = 0.05 kg/(m·s)

Glycerin: μglycerin = 1.49 kg/(m·s)

Let's assume a standard pipe radius of r = 1 cm (0.01 m) and a pipe length of L = 1 m for simplicity.

For water:

ΔPwater = (8 * 0.001 * Q) / (π * [tex](0.01)^{4}[/tex]* 1) = (0.0008 * Q) / (3.1416 * 0.00000001)

= 0.000255 Q.

For SAE 10W oil:

ΔPoil = (8 * 0.05 * Q) / (π * [tex](0.01)^{4}[/tex]* 1) = (0.4 * Q) / (3.1416 * 0.00000001)

= 0.0127 Q.

For glycerin:

ΔPglycerin = (8 * 1.49 * Q) / (π * [tex](0.01)^{4}[/tex]* 1) = (11.92 * Q) / (3.1416 * 0.00000001)

= 0.3793 Q.

From these equations, we can see that the pressure gradient is directly proportional to the flow rate for each fluid.

Conclusion:

Based on the analysis, we can observe the following:

1. Water has the lowest viscosity among the three fluids, resulting in the smallest pressure gradient required for laminar flow.

2. SAE 10W oil has a higher viscosity than water, requiring a larger pressure gradient for the same flow rate.

3. Glycerin has the highest viscosity, leading to the largest pressure gradient needed to maintain laminar flow.

In general, fluids with higher viscosities require larger pressure gradients to achieve the same flow rates, indicating a higher resistance to flow.

It's important to note that these calculations assume laminar flow, which occurs under certain conditions. For higher flow rates or smaller pipe sizes, the flow may transition to turbulent, and different equations would be required to analyze the flow behavior.

To know more about Pressure gradient here

https://brainly.com/question/2530102

#SPJ4

A +6.00 -μC point charge is moving at a constant 8.00×106 m/s in the + y-direction, relative to a reference frame. At the instant when the point charge is at the origin of this reference frame, what is the magnetic-field vectorit produces at the following points.
Part A: x = +.5 m, y = 0 m, z = 0 m
Part B: x = 0 m, y = -.5 m, z = 0 m
Part C: x = 0 m, y = 0 m, z = +.5 m
Part D: x = 0 m, y = -.5 m, z = +.5 m

Answers

The magnetic field vector at point D will be B = Bx i + By j = (-3.83 × 10⁻⁵ T) i + (1.67 × 10⁻⁵ T) j.

Part A: At point A, the magnetic field vector produced by the moving point charge will be in the z-direction and can be calculated using the formula for the magnetic field of a moving point charge. The magnitude of the magnetic field can be calculated using the formula

B = μ₀qv/4πr²,

where μ₀ is the permeability of free space, q is the charge, v is the velocity, and r is the distance from the charge.

Substituting the given values,

we get

B = (4π × 10⁻⁷ T·m/A)(6.00 × 10⁻⁶ C)(8.00 × 10⁶ m/s)/(4π(0.5 m)²)

  = 3.83 × 10⁻⁵ T, directed in the positive z-direction.

Part B: At point B, the magnetic field vector produced by the moving point charge will be in the x-direction and can be calculated using the same formula as in Part A.

Substituting the given values, we get

B = (4π × 10⁻⁷ T·m/A)(6.00 × 10⁻⁶ C)(8.00 × 10⁶ m/s)/(4π(0.5 m)²)

  = 3.83 × 10⁻⁵ T,

directed in the negative x-direction.

Part C: At point C, the magnetic field vector produced by the moving point charge will be in the y-direction and can be calculated using the same formula as in Part A. Substituting the given values, we get

B = (4π × 10⁻⁷ T·m/A)(6.00 × 10⁻⁶ C)(8.00 × 10⁶ m/s)/(4π(0.5 m)²)

  = 3.83 × 10⁻⁵ T,

directed in the positive y-direction.

Part D: At point D, the magnetic field vector produced by the moving point charge will have both x and y components and can be calculated using vector addition of the individual components. The x-component will be the same as in Part B, i.e., Bx = -3.83 × 10⁻⁵ T.

The y-component can be calculated using the formula

By = μ₀qvz/4πr³,

where vz is the velocity component in the z-direction. Substituting the given values, we get

By = (4π × 10⁻⁷ T·m/A)(6.00 × 10⁻⁶ C)(8.00 × 10⁶ m/s)(0.5 m)/(4π(0.5² + 0.5²)³/2)

   = 1.67 × 10⁻⁵ T,

directed in the positive y-direction.

Therefore, the magnetic field vector at point D would be B = Bx i + By j = (-3.83 × 10⁻⁵ T) i + (1.67 × 10⁻⁵ T) j.

To know more about the Point charge, here

https://brainly.com/question/28354110

#SPJ4

describe the physics behind how the universe transitions from an approximately even distribution of matter to the structures we observe today.

Answers

The universe transitions from a homogeneous distribution of matter to the structures we observe today due to the gravitational instability of small density fluctuations generated during inflation. As matter attracts more matter, regions with slightly higher densities grow faster and eventually form galaxies, clusters, and superclusters.

During the inflationary period, quantum fluctuations caused tiny variations in the density of matter. These density fluctuations acted as the seeds of structure formation in the universe. The gravitational attraction between these slightly denser regions led to the formation of larger structures such as galaxies, clusters, and filaments over time. Dark matter played a crucial role in this process, as it provided the necessary gravitational pull to allow the gas to collapse and form stars. The exact details of this process are still under investigation, but it is clear that gravity is the driving force behind the formation of structure in the universe.

Learn more about gravity here:

https://brainly.com/question/31321801

#SPJ11

a lamina occupies the part of the rectangle 0≤x≤2, 0≤y≤4 and the density at each point is given by the function rho(x,y)=2x 5y 6A. What is the total mass?B. Where is the center of mass?

Answers

To find the total mass of the lamina, the total mass of the lamina is 56 units.The center of mass is at the point (My, Mx) = (64/7, 96/7).

A. To find the total mass of the lamina, you need to integrate the density function, rho(x, y) = 2x + 5y, over the given rectangle. The total mass, M, can be calculated as follows:
M = ∫∫(2x + 5y) dA
Integrate over the given rectangle (0≤x≤2, 0≤y≤4).
M = ∫(0 to 4) [∫(0 to 2) (2x + 5y) dx] dy
Perform the integration, and you'll get:
M = 56
So, the total mass of the lamina is 56 units.
B. To find the center of mass, you need to calculate the moments, Mx and My, and divide them by the total mass, M.
Mx = (1/M) * ∫∫(y * rho(x, y)) dA
My = (1/M) * ∫∫(x * rho(x, y)) dA
Mx = (1/56) * ∫(0 to 4) [∫(0 to 2) (y * (2x + 5y)) dx] dy
My = (1/56) * ∫(0 to 4) [∫(0 to 2) (x * (2x + 5y)) dx] dy
Perform the integrations, and you'll get:
Mx = 96/7
My = 64/7
So, the center of mass is at the point (My, Mx) = (64/7, 96/7).

To know more about mass visit :

https://brainly.com/question/28221042

#SPJ11

Other Questions
attractiveness best predicts happiness and social connections for group of answer choices people living in rural settings. people in their 50s. people living in urban settings. people in their 20s. An atom of 110sn has a mass of 109.907858 amu. Calculate the mass defect (deficit) in amu/atom. Use the masses: mass of 1 H atom = 1.007825 amu mass of a neutron = 1.008665 amu 1 amu = 931.5 MeV/c2 Enter your answer in decimal format with four significant figures. en use 2 Using Source 2, which statements best describe contributions of women and African Americans to the war effort during World War II? Select the two correct answers. A African Americans were commanders in integrated combat units. B Women worked in the armaments industry. c Women fought in infantry units. African Americans were captains on warships in the North Atlantic. Women developed strategies to defeat Germany. F African Americans fought in combat units. the role you normally play in society can affect how seriously people take your messages. dopamine can act as a peripheral hormone or as a neurotransmitter. which of these statements would characterize its activity as a hormone? Please help due soonStmt of cash flows problemBalance Sheet for Stand Corp Dec 31st:CashAccounts ReceivableInventoryTotal Current Assets2022$177,000$180,000$100,000$457,0002021$78.000$185.000$100.000$363,000InvestmentsEquipmentAccumulated DeprecTotal Non-Current Liabs$52,000$298,000($106,000)$244.000$74,000$240,000($89.000)$225,000Total Assets$701.000$588.000Current Liabilities$134.000$151.000Common StockRetained Earnings$260.000$307,000$260,000$177,000Total Liabs & Equity$701.000$588,000OTHER INFORMATION:-Investments were sold in 2022 at a loss of $10.000-No equipment was sold in 2022-Cash dividends were paid in 2022-Net Income for 2022 was $160,000REQUIRED:Prepare the Statement of Cash Flows:1. Prepare the Statement of Cash Flows2. Calculate Free Cash Flow fill in the blank. a piece of pie rated at 400 calories is equivalent to _________ calories of thermal energy or __________ joules of mechanical energy. the 6 phases of the system development life cycle are: preliminary investigation; systems analysis; systems design; systems development; systems implementation; and systems maintenance. (true or false) Donna is thinking about buying a house that costs $125,000. If she puts down $25,000 and is able to get a 5 percent mortgage, she wants an estimate of her total monthly housing expenses. The average annual cost for owning a house, beyond the 5% cost of mortgage interest, is 4.09 percent of the value of the house. Not including the opportunity cost of the down payment, amortization on the loan, or any tax benefits from homeownership, what will be her monthly cost the first month?A. $206.25B. $279.60C. $842.71D. $746.43 a certificate contains a unique serial number and must follow which standard that describes the creating of a certificate? Websites are built using the idea of tags to define how a web browser displays the content. An opening tag indicates where the tag starts to takes effect and a closing tag indicates where the tag stops having an effect. Write a statement that correctly assigns annotated Text with the opening tag, followed by the element Text, and then the closing tag. If tagldentifier is 'p' and elementText is 'New paragraph', then annoatatedText is 'Newparagraph'. in order to help sustain culture, which of the following is particularly important? ignatius of loyolas response to the question, "how do i know that i am saved?" what is the difference in allocation between two activity cost pools and three activity cost pools? is the difference in allocation worth using the third activity cost pool .Palace Loan Companys balance sheet at December 31, 2018, reports the following:Preferred stock, $50 par value, 10%, 10,000 shares issued: $500,000Common stock, $0.75 par, 1,000,000 shares issued 750,000Treasury stock, common, 80,000 shares at cost 720,000During 2018, Palace Loan earned net income of $6,300,000. Calculate Palace Loans earnings per common share (EPS) for 2018. (Round EPS to two decimal places.) Assume the number of shares issued and outstanding did not change during the year. Click on the word from the word bank to fill in the blank. In the central dogma, information flows in a specific order. First, the ------- gets--------- into----------. This occurs in the --------------. Second, the RNA gets---------------- into a sequence of-----------------. The ------------------ reads the------------------- RNA every --------------- bases known as--------------. The --------------- RNA carry amino acids to the ribosome to build the-----------. Word Bank:messenger , protein , nucleus DNA , ribosome , codons ,translated ,transcribed ,RNA ,3 ,amino acid ,stransfer The inductance of a solenoid is 75 gl. The solenoid has a length of 0.700 m and a circular cross-section of radius 5.00 cm. What is the winding density (number of turns per unit length of the solenoid? (0-410-71*/A) AX A. 1080 turns B. 865 tums/m C. 327 turm D) 472 turnam E. 104 turns In the month of March, the Baldwin Corporation received and delivered orders of 170,000 units at a price of $15.00 each for revenue of $2.550 million for their product.Baldwin uses the accrual method of accounting and offers 30 day credit terms.By the end of May, Baldwin had collected payments of $2.550 million for the March deliveries.Required:How much of the collected $2.550 million should Baldwin show on the March 31st income statement and how much on the May 31st income statement?a) $2.550 million in March; $0 in May.b) $842 million in March; $1.708 million in May.c) $0 in March; $2.550 million in May.d) $1.275 million in March; $1.275 million in May. Suppose that A was generated uniformly at random from all 8 byte strings. What is the probability that the second binary digit of A is 1? 1/3 1/2 1 1/5 Data analysts prefer to deal with random sampling error rather than statistical bias becauseA. All data analysts are fair peopleB. There is no statistical method for managing statistical biasC. They do not want to be accused of being biased in today's societyD. Random sampling error makes their work more satisfyingE. All of the aboveF. None of the above